User avatar
 
maryadkins
Thanks Received: 641
Atticus Finch
Atticus Finch
 
Posts: 1261
Joined: March 23rd, 2011
 
This post thanked 3 times.
 
 

Q22 - From time to time

by maryadkins Tue Jul 15, 2014 3:01 pm

Here, I'll walk through the whole thing.

The core is:

The mere threat of renewed competition will keep the companies that engaged in "predatory pricing" from raising their prices

-->

Predatory pricing is acceptable

What's the assumption here? Well...how do we know there is going to even be a threat? And where did we ever see a definition of what is "acceptable?" There's no particular reason given for why certain prices are acceptable.

(A) seems pretty good... If we negate it to read, "Some companies that are successful won't necessarily induce competitors to enter the market," then does it destroy our argument? Well, it depends. Are these companies successful or not? What makes a company successful? And wait, do we really need competitors to enter the market? No, we just need the THREAT of it. (A) is out.

(B) doesn't matter. So what? Even if they do it one at a time, the question is whether it's acceptable.

(C) is like (C). Fine, but is it acceptable or not? Is there a threat?

(D) isn't necessary. If we negate it, "It is NOT only the threat of competition or competition that keeps companies from raising prices," is our conclusion destroyed? No, actually, it's stronger! There might be other reasons predatory pricing is fine, too. Great.

(E) ANY pricing practice? Does this seem overly broad? Well, not if we look back at what the argument is saying. It's saying that if the company doesn't raise its prices thanks to the threat of competition, then what it's doing should be acceptable. If we negate (E) to read, "Some pricing practices that don't result in unreasonable prices should not be acceptable," yikes, our argument crumbles, because how do we know this practice is not one of those? The argument doesn't make sense anymore.

(E) is correct.

Hope this helped clarify.

#officialexplanation
 
lsatzen
Thanks Received: 0
Jackie Chiles
Jackie Chiles
 
Posts: 27
Joined: February 25th, 2014
 
 
 

Re: Q22 - From timte to time there is a public outcry

by lsatzen Mon Sep 22, 2014 8:13 pm

Hi All,

I need some help wrapping my head around this problem. I am trying to figure out where I took a wrong turn while answering this question.

I will give a quick run-down of thought process so someone can help me identify where my process led me astray.

Task:
Find an answer choice that must be true in order for the conclusion to be possibly true. We know the argument will be flawed in some way. We know the premises won't fully support the conclusion - find the gap(s) / flaw(s).

Conclusion:
Predatory pricing should be acceptable

Why? Premise:
The mere threat of competition will keep prices down (at least to the extent of not raising beyond unreasonable levels).

Analysis:
My initial gut reaction was: "who cares if prices are down, the author hasn't established an exclusive relationship between prices and acceptable-ness. What about all these other factors that could be relevant to the acceptable-ness of predatory pricing. For example, the prices might be kept down, but what about the the potential moral issue of driving a family-owned business out of business. "

Going into the answer choices, I was looking for an answer that would expose the LSAT 'arrogance' that the one given cause / reason is the only one.

Answer Choices:
I quickly eliminated (A) - (C)
I was down to (D) and (E), and in a rush, I chose (D) because it superficially looked like what I was hunting for.

But coming back to it, is (D) wrong because 1) it doesn't really relate the pricing to the acceptable-ness of predatory pricing (it only seems to reinforce the strength of the premise), 2) the negation of it actually strengthens the argument like Mary pointed out.

(E) is correct, but I had trouble wrapping my head around exactly what it was trying to save because of the double-negative. Is it safe to convert double negatives in a positive frame by canceling out the two negative terms? In this case, I would be inclined to re-word the statement: "any pricing practice that does result in reasonable prices should be acceptable".
 
yunjh2725
Thanks Received: 0
Vinny Gambini
Vinny Gambini
 
Posts: 5
Joined: September 25th, 2014
 
 
 

Re: Q22 - From timte to time there is a public outcry

by yunjh2725 Thu Sep 25, 2014 11:57 pm

@maryadkins

Thank you! Your explanation was so helpful.

But for answer E, isn't it still too extreme? I am confused because a lot of posts here claim that some of these necessary assumption questions' answers are wrong because they are too extreme.

Negation of E: "Not all (or some) pricing practice that does not result in unreasonable prices should be acceptable" As you explained, this predatory pricing can be that "some pricing practice," but what if it's not? I know you said how do we know that it isn't but how do we know that it is?

I am confused because a lot of answers for other necessary questions eliminate these kinds of answers just simply because they are too extreme and there are exceptions. There always seems to be a narrower necessary assumption answer that can eliminate such extreme ones.


Thank you so much in advance!
User avatar
 
maryadkins
Thanks Received: 641
Atticus Finch
Atticus Finch
 
Posts: 1261
Joined: March 23rd, 2011
 
 
 

Re: Q22 - From timte to time there is a public outcry

by maryadkins Sun Sep 28, 2014 11:07 am

"Extreme" answer choices are definitely ones you should be wary of on necessary assumption questions. But if the conclusion is extreme, well, the assumption might be, too!

Think about it: a necessary assumption is something that has to be true for the conclusion to hold. If the conclusion is extreme...it can be making necessary AND extreme assumptions.

Negating (E) means some predatory pricing that leads to unreasonable prices is acceptable. (We know that because that's what negating it tells us. That's the point of negating it.) If that's true, the argument would not hold. Negating (E) destroys the argument.
 
eve.lederman
Thanks Received: 2
Vinny Gambini
Vinny Gambini
 
Posts: 17
Joined: June 03rd, 2014
 
 
 

Re: Q22 - From timte to time there is a public outcry

by eve.lederman Sun Nov 30, 2014 12:27 pm

I got this question right but I'm trying to understand why the other answer choices are wrong. I didn't pick D because I didn't really know what it was doing and I saw that E mentioned the new term from the conclusion. Can anyone explain why D is not necessary? Negated it says that competition is not the only thing that keeps companies from raising prices. So pretty much, D is saying that there are other reasons, including threat, that prevents companies from raising prices, so that is another reason why PP is acceptable.
Is that right?

Any elaboration will be appreciated!
Thanks!
User avatar
 
maryadkins
Thanks Received: 641
Atticus Finch
Atticus Finch
 
Posts: 1261
Joined: March 23rd, 2011
 
 
 

Re: Q22 - From timte to time there is a public outcry

by maryadkins Thu Dec 04, 2014 2:19 pm

That's right...

We don't need (D) to be true because negate it and see what happens:

It is NOT only competition or threat of it that keep companies from raising prices.

Does the argument fall apart?

The argument says that the threat will keep prices from going up too high, just like real competition. What would happen if there were also other things to keep prices from going up? GREAT! The more, the better! If negating an answer choice BOOSTS an argument then it's definitely not a necessary assumption.
 
513852276
Thanks Received: 2
Jackie Chiles
Jackie Chiles
 
Posts: 49
Joined: July 01st, 2014
 
 
 

Re: Q22 - From timte to time there is a public outcry

by 513852276 Mon Jun 01, 2015 1:48 pm

But is it legitimate to assume "pricing practice" only includes predatory pricing? The negation of E is "there are at least one pricing practice which not result in unreasonable prices should not be acceptable". Those pricing practices could be price discrimination, decoy pricing or absorption pricing, as long as "all predatory pricing that does not result in unreasonable prices should be acceptable", the negation of E still not undermine the conclusion. :?:
maryadkins Wrote:"Extreme" answer choices are definitely ones you should be wary of on necessary assumption questions. But if the conclusion is extreme, well, the assumption might be, too!

Think about it: a necessary assumption is something that has to be true for the conclusion to hold. If the conclusion is extreme...it can be making necessary AND extreme assumptions.

Negating (E) means some predatory pricing that leads to unreasonable prices is acceptable. (We know that because that's what negating it tells us. That's the point of negating it.) If that's true, the argument would not hold. Negating (E) destroys the argument.
User avatar
 
maryadkins
Thanks Received: 641
Atticus Finch
Atticus Finch
 
Posts: 1261
Joined: March 23rd, 2011
 
 
 

Re: Q22 - From timte to time there is a public outcry

by maryadkins Tue Jun 02, 2015 10:44 am

The point is that you don't KNOW what the exceptions to (E) are going to be (when you negate it). As long as the pricing discussed in the argument COULD be one of those exceptions, (E) is a necessary assumption.
 
513852276
Thanks Received: 2
Jackie Chiles
Jackie Chiles
 
Posts: 49
Joined: July 01st, 2014
 
 
 

Re: Q22 - From timte to time there is a public outcry

by 513852276 Wed Jun 03, 2015 10:31 am

But is it true that a negation of necessary assumption must undermine the argument? Since the pricing discussed in the argument MAY NOT be one of those exceptions, then a negation of E MAY NOT undermine the argument.

In another word, the necessary assumption is "Any predatory pricing that does not result in unreasonable prices should be acceptable." Instead, "Any pricing practice...", using a more general term, could be sufficient assumption. :?:


maryadkins Wrote:The point is that you don't KNOW what the exceptions to (E) are going to be (when you negate it). As long as the pricing discussed in the argument COULD be one of those exceptions, (E) is a necessary assumption.
User avatar
 
maryadkins
Thanks Received: 641
Atticus Finch
Atticus Finch
 
Posts: 1261
Joined: March 23rd, 2011
 
 
 

Re: Q22 - From timte to time there is a public outcry

by maryadkins Thu Jun 04, 2015 10:09 am

Here is another way to look at it that might make more sense to you.

The entire argument is that since the prices won't rise to unreasonable levels, it should be acceptable. That's the ONLY piece of information they're relying on. So (E) is in this sense a restatement of the exact pieces of the argument, providing the link that the argument is assuming: that if prices aren't unreasonable --> acceptable.

If that principle is not assumed, there is no argument anymore. Because that IS the argument.
 
daijob
Thanks Received: 0
Elle Woods
Elle Woods
 
Posts: 74
Joined: June 02nd, 2015
 
 
 

Re: Q22 - From timte to time there is a public outcry

by daijob Sun Aug 30, 2015 7:23 pm

Hi,

I'm sorry but I'm not sure how negation of D will strengthen the argument...would you please explain it a bit more? :oops:

Thank you
User avatar
 
ohthatpatrick
Thanks Received: 3808
Atticus Finch
Atticus Finch
 
Posts: 4661
Joined: April 01st, 2011
 
 
 

Re: Q22 - From timte to time there is a public outcry

by ohthatpatrick Fri Sep 11, 2015 7:31 pm

The reason our author thinks predatory pricing is cool is that once Wellmart kills off all its competitors, people might be worried that Wellmart will NOW hike its prices to unreasonable levels. After all, now they're the only game in town, so you have to buy your stuff from them.

Our author is saying, don't worry about them jacking up prices once competitors are out of the picture. The mere THREAT of competitors keeps Wellmart from jacking up its prices.

If we negate (D) and learn that OTHER STUFF also keeps Wellmart from jacking up its prices, that would be more support for the author's notion that "you DON'T need to worry about Wellmart jacking up its prices to unreasonable levels."
 
smsotolongo
Thanks Received: 1
Jackie Chiles
Jackie Chiles
 
Posts: 33
Joined: September 21st, 2014
 
This post thanked 1 time.
 
 

Re: Q22 - From timte to time there is a public outcry

by smsotolongo Mon Nov 23, 2015 5:27 pm

Isn't E more of a sufficient assumption rather than a necessary one? I can see from the above how D is not the correct answer when negated. I struggled with this answer and I didn't like either answer and ultimately went with D because I thought E was a sufficient and not a necessary assumption.
 
maria487
Thanks Received: 0
Jackie Chiles
Jackie Chiles
 
Posts: 37
Joined: October 26th, 2015
 
 
 

Re: Q22 - From timte to time there is a public outcry

by maria487 Fri Nov 27, 2015 9:31 pm

smsotolongo Wrote:Isn't E more of a sufficient assumption rather than a necessary one? I can see from the above how D is not the correct answer when negated. I struggled with this answer and I didn't like either answer and ultimately went with D because I thought E was a sufficient and not a necessary assumption.


Hm, I see what you're saying and I do think that this is a stronger answer choice than a lot of other NA answer choices that use weaker language. I think this answer choice is both necessary and sufficient. If you look at the argument, we are going from

Mere threat of competition keeps prices from going up unreasonably --> Pricing practice is acceptable.

Clearly, there's a gap here. What is it? The argument is assuming that the mere threat of competition is enough to prevent a company from raising its prices unreasonably, and that for this reason, predatory pricing is fine. (E) is necessary to for this argument to hold--it's assuming that just because the prices will be kept from going up unreasonably, the predatory pricing is fine--essentially what I spelled out above.

So, I think you are right in that this is both a necessary and sufficient answer :)
 
donghai819
Thanks Received: 7
Elle Woods
Elle Woods
 
Posts: 65
Joined: September 25th, 2015
 
 
 

Re: Q22 - From timte to time there is a public outcry

by donghai819 Wed Jan 13, 2016 10:33 pm

I also picked D. For those who picked D: D would be right if it is written in this way: In addition to competition or the threat of competition, there are other means that keeps companies from raising prices. If we negate it, we will have "Competition or the threat of competition is the only means that keeps companies from raising prices," and thus the argument is destroyed--if it is the only means, we shouldn't allow.
 
contropositive
Thanks Received: 1
Atticus Finch
Atticus Finch
 
Posts: 105
Joined: February 01st, 2015
 
 
 

Re: Q22 - From timte to time there is a public outcry

by contropositive Sun Jan 24, 2016 6:01 pm

I didn't pick D and I didn't negate it either. Couldn't D be eliminated because the word "only" is too strong. Also, it just seems like conclusion booster to say it's the only factor that keeps it from raising prices.

Also, I know that on principle and some other question types when the author prescribes (i.e. should be) we have to look in our answer choice for a recommendation too. Can we apply that method to a necessary/sufficient assumption question?
User avatar
 
ohthatpatrick
Thanks Received: 3808
Atticus Finch
Atticus Finch
 
Posts: 4661
Joined: April 01st, 2011
 
This post thanked 2 times.
 
 

Re: Q22 - From timte to time there is a public outcry

by ohthatpatrick Fri Feb 05, 2016 1:58 pm

I think you mean Premise Booster, since the claim about the threat of competition is the Premise. And you're right. Just because that's the only limiting factor on raising prices that was mentioned doesn't mean that it's the ONLY limiting factor.

Just be careful with language like "couldn't we get rid just because it says something extreme".

As many posters above lamented, the correct answer is just as extreme. ANY pricing practice?

We're allowed to pick some extreme ideas on NA if the author's conclusion is certain. This author's conclusion is CERTAIN about labeling something 'acceptable'. So whatever criteria he laid out in the Evidence must be sufficient, in his thinking, to justify calling the practice acceptable.

That's why we can make a certain move from [stuff we said in premise]- "doesn't result in unreasonable prices" to [thing we're concluding]-"acceptable".

Yes, a prescriptive conclusion (and no premises that are prescriptive) will definitely include prescriptive assumptions. Even more broadly, when you have a new term in the conclusion that hasn't been defined you will definitely need to assume something about how to define that term.

The truth value of this conclusion hinges on whether a practice is acceptable or unacceptable.

Well, to have that debate, we'll need to first define "what do we mean by ACCEPTABLE practice?" The author needs to provide us with the rule HE has in mind, which is (E).
 
ganbayou
Thanks Received: 0
Atticus Finch
Atticus Finch
 
Posts: 213
Joined: June 13th, 2015
 
 
 

Re: Q22 - From timte to time there is a public outcry

by ganbayou Wed Aug 03, 2016 9:04 pm

So...does D strengthen the argument or it is just not nec. to assume?
At first I thought E is sufficient assumption so eliminated it...can sufficient ass. be the correct ans. if there is no other good ans. in nec. assumption question?
If negation strengthen the argument, does it weaken or st. the argument?
I simply thought D is P booster because in the premise it says "the mere threat" and D says "It is only..."
Did I understand it correctly?? :(
 
andrewgong01
Thanks Received: 61
Atticus Finch
Atticus Finch
 
Posts: 289
Joined: October 31st, 2016
 
 
 

Re: Q22 - From timte to time there is a public outcry

by andrewgong01 Thu Jun 01, 2017 6:35 pm

I understand now the above negation of "D" but can someone point out where I went wrong with this version of seeing "D" :

D: If companies raise prices then there is neither threat of competition nor competition.
Contrapositive: If there is competition or the threat of it then companies won't raise prices
Negated Contrapositive : If there is competition or the threat of it companies will raise prices

I am not sure what I am missing here but then my negated contrapositive would then seem to also weaken the argument because it is saying competition/the threat of it also induce price rises and then the argument falls apart

An issue I had with "E" is that it sounds like a SA and the way I attacked it on the test was that , okay fine I will take it to be true but why does the author need to assume "any pricing practice". For example, say we have controlled/fix pricing and that won't lead to unreasonable prices. But how do we know the author here would support and say controlled/fix prices is good too? That said had "E" said "Predatory Pricing that does not result in unreasonable prices should be acceptable" would be too blatant and giving away the answer

Edit: I now see that if "E" was negated as Any pricing practice that does not result in unreasonable prices should be unacceptable" does destroy the argument and convinces me that way but I still don't see why my earlier example of the fixed pricing would not cast doubt on "E"'s validity
User avatar
 
ohthatpatrick
Thanks Received: 3808
Atticus Finch
Atticus Finch
 
Posts: 4661
Joined: April 01st, 2011
 
 
 

Re: Q22 - From timte to time there is a public outcry

by ohthatpatrick Fri Jun 02, 2017 2:34 pm

D doesn't strengthen either.

We already know that threat of competition can keep a company from raising prices (from the Premise).

(D) is telling us whether or not there are OTHER things that keep prices low. Who cares whether there are or aren't. They would be irrelevant to the conversation we're having.

If I said,
"Chewing gum will help increase your LSAT score. After all, chewing gum helps to reduce your stress level.

The gap we care about is whether "reducing stress --> increase LSAT score".

Would it make any difference if I said, "Hey, other things can ALSO reduce your stress"